LSAT and Law School Admissions Forum

Get expert LSAT preparation and law school admissions advice from PowerScore Test Preparation.

 Administrator
PowerScore Staff
  • PowerScore Staff
  • Posts: 8917
  • Joined: Feb 02, 2011
|
#81371
Complete Question Explanation

The correct answer choice is (B).

Answer choice (A):

Answer choice (B): This is the correct answer choice.

Answer choice (C):

Answer choice (D):

Answer choice (E):

This explanation is still in progress. Please post any questions below!
 Etsevdos
  • Posts: 62
  • Joined: Oct 22, 2017
|
#40911
I was between B and E. line 22 : "...somewhat increases quantity....doesn't this suggest that B is not correct? I found this question very difficult because of this line. Thanks
 James Finch
PowerScore Staff
  • PowerScore Staff
  • Posts: 943
  • Joined: Sep 06, 2017
|
#40971
Hi Etsevdos,

That's a good question! To answer it, we have to look at the context of the part you quoted. The full quote is:

"While preserving terminological distinctions somewhat increases the quality of the information extracted from medieval documents concerning women medical practitioners, scholars must also reopen the whole question of why documentary evidence for women medical practitioners comprises such a tiny fraction of the evidence historians of medieval medicine usually present." (emphasis mine)

When put in full context, this sentence and the rest of the paragraph that follows, as well as the third paragraph, all are presenting quantitative arguments about the traditional methodology undercounting medieval women medical practitioners. The argument made in these two paragraphs is that limiting research to medieval documents leads to ignoring the majority of female practitioners, as they worked in more marginal fields that would not necessarily be memorialized in the kinds of documents traditionally studied. Only in the last paragraph do we see a switch to a qualitative argument.

The last paragraph delves into the potential qualitative benefits ("social context, etc.") of introducing cross-disciplinary approaches to these studies of medieval medical practitioners. This is also what makes (B) the correct answer choice over (E). (E) deals only with one part of the author's argument, a part given comparatively few words compared to the much more elaborate quantitative argument. (B) contains both the qualitative and quantitative elements of the author's overall argument, thus capturing both parts of the main point of the passage.

Hope this helps!
 lunsandy
  • Posts: 61
  • Joined: Oct 14, 2017
|
#41567
Hi Powerscore,

Can someone speak to why C is incorrect? I was stuck between B and C during the exam and end up choosing C. I didn't like the quantity and quality aspect of answer B, but as James pointed out from the previous post, I now see why B is the better answer.

When C says "limitations of the historical record" I thought it was talking about the main idea of historians using the method of narrowly defining who is considered a "woman medical practitioner."

Thanks a lot!
 nicholaspavic
PowerScore Staff
  • PowerScore Staff
  • Posts: 271
  • Joined: Jun 12, 2017
|
#42099
Hi lunsandy,

Can you tell me where in the passage you are specifically connecting those two ideas? I do not see any support for that inference. Thanks for the question and let us know.
 T.B.Justin
  • Posts: 194
  • Joined: Jun 01, 2018
|
#61356
The author suggests what future studies possibly can do to improve on the previous information that historians uncovered concerning women's medical practice roles in medieval Europe, and the correct answer choice goes on to make a hypothetical that, "if they did those things, it would improve the quantity and quality of the previous information."

Going from can possibly improve something (in the passage) to would improve something (best expressed main point) is the problem I have with the correct answer choice.

My thinking is just because those things can improve the quantity and quality of the information does not necessarily guarantee they would.

I think answer choice "E" is solely focused on the last paragraph, but I don't have an issue with an author expressing the main point of the passage in one paragraph.

I have a problem with the usage of "more information" in this answer choice. It doesn't say if the information will be better, the same, or worse than previous information. Basically it leaves out quality, and answer choice "B" addresses both quantity and quality.

My question is the test makers thinking for choosing "would" instead of "could" in the correct answer choice "B"
 Brook Miscoski
PowerScore Staff
  • PowerScore Staff
  • Posts: 418
  • Joined: Sep 13, 2018
|
#62686
Justin,

Although your wording is a bit unclear, I believe you are asking why there is justification for the word "would" instead of "could."

In line 43, the author states "The advantage of broadening the scope of such studies is immediately apparent..." The author is not arguing that there might be a benefit, the author is arguing that there's a clear and immediate benefit.

The later "might" is in reference to what people could then additionally do with that benefit, it is not about whether there is uncertainty as to the benefit.
 T.B.Justin
  • Posts: 194
  • Joined: Jun 01, 2018
|
#62696
Hey Brook,

Ok, so it would improve, and with that, it is then "left up in the air," of the perhaps endless possibilities from that improvement, got it.
 Adeline
  • Posts: 23
  • Joined: Dec 31, 2018
|
#65910
Hi,

I have a question about Question 22. I don't understand why is E incorrect. Could you further explain it?
I hesitated between B and E. E seems very similar to B in the sense that both of them talk about improvement of research methods. E is also presented in the last paragraph as an author's suggestion of improvement for this field of studies, which looks very attractive to be the correct answer. Is E incorrect because it is too narrow to illustrate the main point of the whole passage?

Thank you.
 George George
PowerScore Staff
  • PowerScore Staff
  • Posts: 48
  • Joined: Jun 07, 2019
|
#66205
@Adeline This is good Q! I agree with you that answer (E) points to the author's recommendation in the last paragraph, and I had a hard time debunking this as the correct answer. I believe the key to understanding why (B) is the main point and not (E) is to look at the passage's structure. The author's recommendations only come in after a critique or rebuttal of the mainstream historians. The passage is not just about one specific recommendation, but also about the fact that the current data about medieval midwives is lacking/insufficient. The takeaway here is that, on Main Point Qs, some wrong answers will be true, but not the main idea. In other words, you'll have to dodge answers that would be correct on a Must Be True Q, but aren't the main idea. Another takeaway is that when the author rebuts a commonly held viewpoint, that rebuttal is also part of the main point. (B) includes more than (E) in this respect.

Get the most out of your LSAT Prep Plus subscription.

Analyze and track your performance with our Testing and Analytics Package.